Question

For each of the following production functions, solve for the marginal products of each input and marginal rate of substitution.

For each of the following production functions, solve for the marginal products of each input and marginal rate of substitution. Then answer the following for each: does this production function exhibit diminishing marginal product of labour? Does this production function exhibit diminishing marginal product of capital? Does this production function exhibit constant, decreasing, or increasing returns to scale? Show all your work.

(a) \(Q=L+K\)

(b) \(Q=2 L^{2}+K^{2}\)

(c) \(Q=L^{1 / 2} K^{1 / 2}\)

0 0
Add a comment Improve this question Transcribed image text
✔ Recommended Answer
Answer #1

(a) \(Q=L+K\)

Marginal product of labor \(L\) is given by \(\frac{\partial Q}{\partial L}=\frac{\partial(L+K)}{\partial L}=1\)

Marginal product of labor \(\mathrm{K}\) is given by \(\frac{\partial Q}{\partial K}=\frac{\partial(L+K)}{\partial K}=1\)

Marginal rate of substitution is (Marginal product of labor L)/(Marginal product of capital \(\mathrm{K}\) ) \(=\frac{1}{1}=1\) So,marginal rate of substitution=1.

No this function does not exhibit diminishing marginal product of labor because marginal product of labor

is constant \(\frac{\partial^{2} Q}{\partial L^{2}}=\frac{\partial^{2}(L+K)}{\partial L^{2}}=0\)

No this function does not exhibit diminishing marginal product of capital because marginal product of capital is constant and \(\frac{\partial^{2} Q}{\partial K^{2}}=\frac{\partial^{2}(L+K)}{\partial K^{2}}=0\)

\(Q=L+K\)

When both L and \(K\) input are increased by same \(\alpha\) where \(\alpha>0\) then new \(Q\) will be such that:

\(Q=\alpha L+\alpha K\)

\(Q=\alpha(L+K)\)

Since the power of \(\alpha\) is 1 this imply \(Q=L+K\) exhibit constant returns to scale because output increases in same proportion as increase in inputs.

(b) \(Q=2 L^{2}+K^{2}\)

Marginal product of labor \(L\) is given by \(\frac{\partial Q}{\partial L}=\frac{\partial\left(2 L^{2}+K^{2}\right)}{\partial L}=2 * 2 L=4 L\)

Marginal product of labor \(\mathrm{K}\) is given by \(\frac{\partial Q}{\partial K}=\frac{\partial\left(2 L^{2}+K^{2}\right)}{\partial K}=2 K\)

Marginal rate of substitution is (Marginal product of labor L)/(Marginal product of capital \(\mathrm{K}=\frac{4 L}{2 K}=\frac{2 L}{K}\)

No this function does not exhibit diminishing marginal product of labor because when L is 1 marginal product of labor is \(4\left(4^{*} 1\right),\) L increase to 2 units then marginal product of labor is \(8\left(4^{*} 2\right),\) L increase to 3 units then marginal product of labor is \(12\left(4^{*} 3\right)\) which imply this function does not exhibit diminishing marginal product of labor as labor units increase marginal product of labor also increase.

No this function does not exhibit diminishing marginal product of capital because when \(\mathrm{K}\) is 1 marginal product of capital is \(2\left(2^{*} 1\right), \mathrm{K}\) increase to 2 units then marginal product of capital is \(4\left(2^{*} 2\right), \mathrm{K}\) increase to 3 units then marginal product of capital is \(6\left(2^{*} 3\right)\) which imply this function does not exhibit diminishing marginal product of capital as capital unit increase marginal product of capital also increase.

\(Q=2 L^{2}+K^{2}\)

When both \(L\) and \(K\) input are increased by same \(\alpha\) where \(\alpha>0\) then new \(Q\) will be such that:

\(Q=2(\alpha L)^{2}+(\alpha K)^{2}\)

\(Q=2 \alpha^{2} L^{2}+\alpha^{2} K^{2}\)

\(Q=\alpha^{2}\left(2 L^{2}+K^{2}\right)\)

Since the power of \(\alpha\) is greater than 1 this imply \(Q=2 L^{2}+K^{2}\) exhibit increasing returns to scale because output increases in more proportion than there is increase in inputs.

(c) \(Q=L^{\frac{1}{2}} K^{\frac{1}{2}}\)

Marginal product of labor L is given by \(\frac{\partial Q}{\partial L}=\frac{\partial\left(L^{\frac{1}{2}} K^{\frac{1}{2}}\right)}{\partial L}=\frac{1}{2} * \frac{K^{\frac{1}{2}}}{L^{\frac{1}{2}}}=\frac{K^{\frac{1}{2}}}{2 L^{\frac{1}{2}}}\)

Marginal product of labor \(\mathrm{K}\) is given by \(\frac{\partial Q}{\partial K}=\frac{\partial\left(L^{\frac{1}{2}} K^{\frac{1}{2}}\right)}{\partial K}=\frac{1}{2} * \frac{L^{\frac{1}{2}}}{K^{\frac{1}{2}}}=\frac{L^{\frac{1}{2}}}{2 K^{\frac{1}{2}}}\)

Marginal rate of substitution is (Marginal product of labor L)/(Marginal product of capital K)= \(\frac{\frac{K}{2 L}}{\frac{L}{2 K}}=\frac{K * 2 K}{2 L * L}=\frac{K}{L}\). So,marginal rate of substitution \(=\frac{K}{L}\)

Yes this function does exhibit diminishing marginal product of labor because \(\frac{\partial^{2} Q}{\partial L^{2}}=\frac{\partial^{2}\left(L^{\frac{1}{2}} K^{\frac{1}{2}}\right)}{\partial L^{2}}=\frac{-1 * K^{\frac{1}{2}}}{4 L^{\frac{3}{2}}}=\frac{-K^{\frac{1}{2}}}{4 L^{\frac{3}{2}}}\) is negative which means marginal product of labor fall when

Lincreases by 1 unit.

Yes this function does exhibit diminishing marginal product of capital because \(\frac{\partial^{2} Q}{\partial L^{2}}=\frac{\partial^{2}\left(L^{\frac{1}{2}} K^{\frac{1}{2}}\right)}{\partial L^{2}}=\frac{-1 * L^{\frac{1}{2}}}{4 K^{\frac{3}{2}}}=\frac{-L^{\frac{1}{2}}}{4 K^{\frac{3}{2}}}\) is negative which means marginal product of capital fall

when \(\mathrm{K}\) increases by 1 unit.

\(Q=L^{\frac{1}{2}} K^{\frac{1}{2}}\)

When both \(L\) and \(K\) input are increased by same \(\alpha\) where \(\alpha>0\) then new \(Q\) will be such that:

\(Q=(\alpha L)^{\frac{1}{2}}(\alpha K)^{\frac{1}{2}}\)

\(Q=\alpha^{\frac{1}{2}} L^{\frac{1}{2}} \alpha^{\frac{1}{2}} K^{\frac{1}{2}}\)

\(Q=\alpha L^{\frac{1}{2}} K^{\frac{1}{2}}\)

Since the power of \(\alpha\) is 1 this imply \(Q=L^{\frac{1}{2}} K^{\frac{1}{2}}\) exhibit constant returns to scale because output increases in same proportion as increase in inputs.

Add a comment
Know the answer?
Add Answer to:
For each of the following production functions, solve for the marginal products of each input and marginal rate of substitution.
Your Answer:

Post as a guest

Your Name:

What's your source?

Earn Coins

Coins can be redeemed for fabulous gifts.

Similar Homework Help Questions
  • 2. For each of the following production functions: Q = 3L2 +6K4 Q= V25KL Q =...

    2. For each of the following production functions: Q = 3L2 +6K4 Q= V25KL Q = 5V1 + 101K a. Find the marginal product of each input, i.e. MP, and MPx: b. Determine whether the production function exhibits diminishing marginal returns to each input. c. Find the marginal rate of technical substitution, MRTSLK and discuss how it changes as the firm uses more L and less K, holding output constant. d. Determine whether the production function exhibits constant, increasing or...

  • Consider the production function given by y = f(L,K) = L^(1/2) K^(1/3) , where y is...

    Consider the production function given by y = f(L,K) = L^(1/2) K^(1/3) , where y is the output, L is the labour input, and K is the capital input. (a) Does this exhibit constant, increasing, or decreasing returns to scale? (b) Suppose that the firm employs 9 units of capital, and in the short-run, it cannot change this amount. Then what is the short-run production function? (c) Determine whether the short-run production function exhibits diminishing marginal product of labour. (d)...

  • 3. For each of the following production functions, graph a typical isoquant and determine whether the marginal rate of...

    3. For each of the following production functions, graph a typical isoquant and determine whether the marginal rate of technical substitution of labor for capital (MRTS ) is diminishing, constant, increasing, or none of these. a. Q=LK b. Q=LVK c. Q=L*K13 d. Q = 3L +K e. Q = min{3L, K} Show transcribed image text 3. For each of the following production functions, graph a typical isoquant and determine whether the marginal rate of technical substitution of labor for capital...

  • 3) Consider the production function ? = 6? 0.3? 0.6 . The marginal products are ???...

    3) Consider the production function ? = 6? 0.3? 0.6 . The marginal products are ??? = 1.8? −0.7? 0.6 and ??? = 3.6? 0.3? −0.4 . a. In the short run assume that capital is fixed at ? = 10. Derive formulas for the short-run Total Product (TP), Average Product (APL), and Marginal Product (MPL). Graph these three functions. b. In the long run, capital is not fixed. Graph the isoquant for ? = 6. Identify and label three...

  • The production function q = k0.620.5 exhibits: a. increasing returns to scale and diminishing marginal products...

    The production function q = k0.620.5 exhibits: a. increasing returns to scale and diminishing marginal products for both k and 1. b. increasing returns to scale and diminishing marginal product for 1 only. c. increasing returns to scale but no diminishing marginal productivities. d. decreasing returns to scale.

  • the second question In Example 6.4 wheat is produced according to the production function: q=100(k0.6 0.4)...

    the second question In Example 6.4 wheat is produced according to the production function: q=100(k0.6 0.4) Beginning with a capital input of 4 and a labor input of 49, show that the marginal product of labor and the marginal product of capital are both decreasing (Round responses to two decimal places.) The MPK at 5 units of capital is 156.12 The MP at 6 units of capital is 144.02 The MP at 50 units of labor is 8.84 The MP...

  • Determine whether the following production functions exhibit constant, increasing, or decreasing returns to scale. L, K,...

    Determine whether the following production functions exhibit constant, increasing, or decreasing returns to scale. L, K, and H are inputs and Q is the output in each production function. Initially, set each input = 100 and determine the output. Then increase each input by 2% and determine the corresponding output to see if constant, increasing, or decreasing returns to scale occur.                             (a) Q = 0.5L + 2K + 40H                                                                                                                                                               (b) Q = 3L + 10K +...

  • The marginal product of labour (MPL) at a particular labour input level (L) corresponds to the...

    The marginal product of labour (MPL) at a particular labour input level (L) corresponds to the __ on the total product of labour (TPL) curve. O area under the curve between 0 and a particular level of L area above the curve between 0 and a particular level of L slope of the ray from the (0,0) origin to the particular level of L slope of a tangent line at the particular level of L The average product of labour...

  • 2. For the following Cobb-Douglas production function, q = f(L,K) = _0.45 0.7 a. Derive expressions...

    2. For the following Cobb-Douglas production function, q = f(L,K) = _0.45 0.7 a. Derive expressions for marginal product of labor and marginal product of capital, MP, and MPK. b. Derive the expression for marginal rate of technical substitution, MRTS. C. Does this production function display constant, increasing, or decreasing returns to scale? Why? d. By how much would output increase if the firm increased each input by 50%?

  • Given the following long run production and cost functions: q=LPK1/4 C = 12L +4K (A) What...

    Given the following long run production and cost functions: q=LPK1/4 C = 12L +4K (A) What input has diminishing marginal returns? (B) Does this production function display increasing, decreasing or constant returns to scale? (C) What is this firm's expansion path assuming input prices do not change? Clearly type out your answer to parts (A), (B) and (C) in the space provided. Retain all of your handwritten work for this question to be uploaded separately after you have completed the...

ADVERTISEMENT
Free Homework Help App
Download From Google Play
Scan Your Homework
to Get Instant Free Answers
Need Online Homework Help?
Ask a Question
Get Answers For Free
Most questions answered within 3 hours.
ADVERTISEMENT
ADVERTISEMENT
ADVERTISEMENT